LSAT and Law School Admissions Forum

Get expert LSAT preparation and law school admissions advice from PowerScore Test Preparation.

User avatar
 Dave Killoran
PowerScore Staff
  • PowerScore Staff
  • Posts: 5852
  • Joined: Mar 25, 2011
|
#45457
Complete Question Explanation
(The complete setup for this game can be found here: lsat/viewtopic.php?t=13934)

The correct answer choice is (A)

Since L has two symptoms, and L must be different than the other two-symptom illness—which is J—L cannot have H and S. That means that L must have F and either H or S. Since L must have F, answer choice (A) is correct.
 CRicksy
  • Posts: 2
  • Joined: Oct 19, 2018
|
#59677
Hello Powerscore,

In the explanation for question #9, it states that L must be different than the other two-symptom illnesses. I was wondering where that comes from in the question?

Thanks!
 Adam Tyson
PowerScore Staff
  • PowerScore Staff
  • Posts: 5153
  • Joined: Apr 14, 2011
|
#59714
That's the situation described in the question, CRicksy - L must be different from every other illness. It cannot have a combination of symptoms that matches any other illness. In other words, it must be unique.
 g_lawyered
  • Posts: 211
  • Joined: Sep 14, 2020
|
#86727
Hi P.S.,
This wording of this question definitely tripped me up. Questions states "If illness L is characterized by a combination of symptoms different from any of the other illnesses". I misunderstood this to mean that L couldn't share symptoms with the rest of the illness meaning: K, M, and N. Because of rule 3, I assumed J and L could only share either H/S and I forgot about the Numerical Distribution that L must have 2 symptoms. I drew out that possibility and did follow rule 3 that L must have H or S. I didn't conclude it must have F. Which is why I got question incorrect.
After reading the explanation, it's clear to that L must have F (to follow the numerical distribution of L having 2 symptoms). Which is why answer A is correct.
But can someone explain to me how/when we're suppose to interpret the wording of the question (in quotations) to mean that L can only share 1 symptom with J. I'm sure there will be more LG questions with this tricky wording and I don't want to keep missing the question because of it.

Thanks in advance!
User avatar
 Ryan Twomey
PowerScore Staff
  • PowerScore Staff
  • Posts: 141
  • Joined: Mar 04, 2021
|
#86746
Hey GGIBA,

So how was your setup with this game? Did you have every deduction that Dave drew in the setup? That would be the first place to start.

The question stem is difficult wording, but we also know from our setup that it is not possible for L to have 0 symptoms in common with any of the other illnesses. So you can assume that the question is meaning that L will not have the exact same symptoms as any of the other illnesses. Often times, you can use your setup to help you determine what the language in the question stem means. But also, the word "combination" can lead you to the direction to assume they mean a "different combination" rather than having "none in common."

L and J both have two symptoms based off a series of deductions from your setup, namely the first four rules give us that. So since you know J has H and S from your rules, then you know L has to have H/S and then F, so it does not exactly match J.

I hope all this helps.

Good luck with your studying and keep at it!

Best,
Ryan

Get the most out of your LSAT Prep Plus subscription.

Analyze and track your performance with our Testing and Analytics Package.